Signed measure of uncountable set

The name of the pictureThe name of the pictureThe name of the pictureClash Royale CLAN TAG#URR8PPP











up vote
1
down vote

favorite












I have a question and hope some of you can help me :)



Consider a signed measure $nu$ on $(Omega, bfA)$ and let be $P_i in bfA$ positive sets, such that $ forall B subset P_i: nu(B) geq0 $. Then we state that $bigcup_i in I P_i$ is also a positive set.



If $I$ is countable ( $|I| leq |mathbbN|$) this seems to be true.
To show this I considered an arbitrary $M in bigcup_i in I P_i $. I know that there exists $M_i := M cap P_i subset P_i$. So we get $nu(M) = nu left( bigcup_i in IM_i right) = sum_i in I nu(M_i) geq 0$.



The last $=$ follows due to sigma-additivity of $nu$ and the $geq$ follows due to $M_i subset P_i$ and $P_i$ is positive.



Now I wonder, what happens, if $I$ is uncountable ( $|I| > |mathbbN|$). I think there must be a counterexample, because the sigma-additivity doesn't hold anymore, but I can't find one.



Does anybody have a counterexample for me, or is my assumption wrong, and one can proof, that the statement is still true for overcountable I's.



Thanks a lot!










share|cite|improve this question





















  • Check the first answer given in math.stackexchange.com/questions/1605076/…
    – UserS
    2 hours ago















up vote
1
down vote

favorite












I have a question and hope some of you can help me :)



Consider a signed measure $nu$ on $(Omega, bfA)$ and let be $P_i in bfA$ positive sets, such that $ forall B subset P_i: nu(B) geq0 $. Then we state that $bigcup_i in I P_i$ is also a positive set.



If $I$ is countable ( $|I| leq |mathbbN|$) this seems to be true.
To show this I considered an arbitrary $M in bigcup_i in I P_i $. I know that there exists $M_i := M cap P_i subset P_i$. So we get $nu(M) = nu left( bigcup_i in IM_i right) = sum_i in I nu(M_i) geq 0$.



The last $=$ follows due to sigma-additivity of $nu$ and the $geq$ follows due to $M_i subset P_i$ and $P_i$ is positive.



Now I wonder, what happens, if $I$ is uncountable ( $|I| > |mathbbN|$). I think there must be a counterexample, because the sigma-additivity doesn't hold anymore, but I can't find one.



Does anybody have a counterexample for me, or is my assumption wrong, and one can proof, that the statement is still true for overcountable I's.



Thanks a lot!










share|cite|improve this question





















  • Check the first answer given in math.stackexchange.com/questions/1605076/…
    – UserS
    2 hours ago













up vote
1
down vote

favorite









up vote
1
down vote

favorite











I have a question and hope some of you can help me :)



Consider a signed measure $nu$ on $(Omega, bfA)$ and let be $P_i in bfA$ positive sets, such that $ forall B subset P_i: nu(B) geq0 $. Then we state that $bigcup_i in I P_i$ is also a positive set.



If $I$ is countable ( $|I| leq |mathbbN|$) this seems to be true.
To show this I considered an arbitrary $M in bigcup_i in I P_i $. I know that there exists $M_i := M cap P_i subset P_i$. So we get $nu(M) = nu left( bigcup_i in IM_i right) = sum_i in I nu(M_i) geq 0$.



The last $=$ follows due to sigma-additivity of $nu$ and the $geq$ follows due to $M_i subset P_i$ and $P_i$ is positive.



Now I wonder, what happens, if $I$ is uncountable ( $|I| > |mathbbN|$). I think there must be a counterexample, because the sigma-additivity doesn't hold anymore, but I can't find one.



Does anybody have a counterexample for me, or is my assumption wrong, and one can proof, that the statement is still true for overcountable I's.



Thanks a lot!










share|cite|improve this question













I have a question and hope some of you can help me :)



Consider a signed measure $nu$ on $(Omega, bfA)$ and let be $P_i in bfA$ positive sets, such that $ forall B subset P_i: nu(B) geq0 $. Then we state that $bigcup_i in I P_i$ is also a positive set.



If $I$ is countable ( $|I| leq |mathbbN|$) this seems to be true.
To show this I considered an arbitrary $M in bigcup_i in I P_i $. I know that there exists $M_i := M cap P_i subset P_i$. So we get $nu(M) = nu left( bigcup_i in IM_i right) = sum_i in I nu(M_i) geq 0$.



The last $=$ follows due to sigma-additivity of $nu$ and the $geq$ follows due to $M_i subset P_i$ and $P_i$ is positive.



Now I wonder, what happens, if $I$ is uncountable ( $|I| > |mathbbN|$). I think there must be a counterexample, because the sigma-additivity doesn't hold anymore, but I can't find one.



Does anybody have a counterexample for me, or is my assumption wrong, and one can proof, that the statement is still true for overcountable I's.



Thanks a lot!







measure-theory signed-measures






share|cite|improve this question













share|cite|improve this question











share|cite|improve this question




share|cite|improve this question










asked 2 hours ago









pcalc

975




975











  • Check the first answer given in math.stackexchange.com/questions/1605076/…
    – UserS
    2 hours ago

















  • Check the first answer given in math.stackexchange.com/questions/1605076/…
    – UserS
    2 hours ago
















Check the first answer given in math.stackexchange.com/questions/1605076/…
– UserS
2 hours ago





Check the first answer given in math.stackexchange.com/questions/1605076/…
– UserS
2 hours ago











2 Answers
2






active

oldest

votes

















up vote
3
down vote



accepted










Let $[0,1]$ be equipped with negative Lebesgue measure.



All singletons are positive sets but the union of all singletons is not.






share|cite|improve this answer




















  • Aaah thanks! I've already thought about this example, but rejected it, because I thought I want that it has to hold for ALL subsets and their union - what obviously is nonsense. Thanks a lot!
    – pcalc
    2 hours ago










  • You are welcome.
    – drhab
    2 hours ago

















up vote
1
down vote













Consider $(Bbb R,textLebesgue)$, $I=[0,1]$ and $P_i=i$ with the measure $nu(A)=-int_A e^-x^2sin x,dx$.






share|cite|improve this answer




















  • Thank you! It seems as I have overseen quite a lot of good examples.
    – pcalc
    2 hours ago










Your Answer




StackExchange.ifUsing("editor", function ()
return StackExchange.using("mathjaxEditing", function ()
StackExchange.MarkdownEditor.creationCallbacks.add(function (editor, postfix)
StackExchange.mathjaxEditing.prepareWmdForMathJax(editor, postfix, [["$", "$"], ["\\(","\\)"]]);
);
);
, "mathjax-editing");

StackExchange.ready(function()
var channelOptions =
tags: "".split(" "),
id: "69"
;
initTagRenderer("".split(" "), "".split(" "), channelOptions);

StackExchange.using("externalEditor", function()
// Have to fire editor after snippets, if snippets enabled
if (StackExchange.settings.snippets.snippetsEnabled)
StackExchange.using("snippets", function()
createEditor();
);

else
createEditor();

);

function createEditor()
StackExchange.prepareEditor(
heartbeatType: 'answer',
convertImagesToLinks: true,
noModals: false,
showLowRepImageUploadWarning: true,
reputationToPostImages: 10,
bindNavPrevention: true,
postfix: "",
noCode: true, onDemand: true,
discardSelector: ".discard-answer"
,immediatelyShowMarkdownHelp:true
);



);













 

draft saved


draft discarded


















StackExchange.ready(
function ()
StackExchange.openid.initPostLogin('.new-post-login', 'https%3a%2f%2fmath.stackexchange.com%2fquestions%2f2963260%2fsigned-measure-of-uncountable-set%23new-answer', 'question_page');

);

Post as a guest






























2 Answers
2






active

oldest

votes








2 Answers
2






active

oldest

votes









active

oldest

votes






active

oldest

votes








up vote
3
down vote



accepted










Let $[0,1]$ be equipped with negative Lebesgue measure.



All singletons are positive sets but the union of all singletons is not.






share|cite|improve this answer




















  • Aaah thanks! I've already thought about this example, but rejected it, because I thought I want that it has to hold for ALL subsets and their union - what obviously is nonsense. Thanks a lot!
    – pcalc
    2 hours ago










  • You are welcome.
    – drhab
    2 hours ago














up vote
3
down vote



accepted










Let $[0,1]$ be equipped with negative Lebesgue measure.



All singletons are positive sets but the union of all singletons is not.






share|cite|improve this answer




















  • Aaah thanks! I've already thought about this example, but rejected it, because I thought I want that it has to hold for ALL subsets and their union - what obviously is nonsense. Thanks a lot!
    – pcalc
    2 hours ago










  • You are welcome.
    – drhab
    2 hours ago












up vote
3
down vote



accepted







up vote
3
down vote



accepted






Let $[0,1]$ be equipped with negative Lebesgue measure.



All singletons are positive sets but the union of all singletons is not.






share|cite|improve this answer












Let $[0,1]$ be equipped with negative Lebesgue measure.



All singletons are positive sets but the union of all singletons is not.







share|cite|improve this answer












share|cite|improve this answer



share|cite|improve this answer










answered 2 hours ago









drhab

91.6k542124




91.6k542124











  • Aaah thanks! I've already thought about this example, but rejected it, because I thought I want that it has to hold for ALL subsets and their union - what obviously is nonsense. Thanks a lot!
    – pcalc
    2 hours ago










  • You are welcome.
    – drhab
    2 hours ago
















  • Aaah thanks! I've already thought about this example, but rejected it, because I thought I want that it has to hold for ALL subsets and their union - what obviously is nonsense. Thanks a lot!
    – pcalc
    2 hours ago










  • You are welcome.
    – drhab
    2 hours ago















Aaah thanks! I've already thought about this example, but rejected it, because I thought I want that it has to hold for ALL subsets and their union - what obviously is nonsense. Thanks a lot!
– pcalc
2 hours ago




Aaah thanks! I've already thought about this example, but rejected it, because I thought I want that it has to hold for ALL subsets and their union - what obviously is nonsense. Thanks a lot!
– pcalc
2 hours ago












You are welcome.
– drhab
2 hours ago




You are welcome.
– drhab
2 hours ago










up vote
1
down vote













Consider $(Bbb R,textLebesgue)$, $I=[0,1]$ and $P_i=i$ with the measure $nu(A)=-int_A e^-x^2sin x,dx$.






share|cite|improve this answer




















  • Thank you! It seems as I have overseen quite a lot of good examples.
    – pcalc
    2 hours ago














up vote
1
down vote













Consider $(Bbb R,textLebesgue)$, $I=[0,1]$ and $P_i=i$ with the measure $nu(A)=-int_A e^-x^2sin x,dx$.






share|cite|improve this answer




















  • Thank you! It seems as I have overseen quite a lot of good examples.
    – pcalc
    2 hours ago












up vote
1
down vote










up vote
1
down vote









Consider $(Bbb R,textLebesgue)$, $I=[0,1]$ and $P_i=i$ with the measure $nu(A)=-int_A e^-x^2sin x,dx$.






share|cite|improve this answer












Consider $(Bbb R,textLebesgue)$, $I=[0,1]$ and $P_i=i$ with the measure $nu(A)=-int_A e^-x^2sin x,dx$.







share|cite|improve this answer












share|cite|improve this answer



share|cite|improve this answer










answered 2 hours ago









Saucy O'Path

4,649424




4,649424











  • Thank you! It seems as I have overseen quite a lot of good examples.
    – pcalc
    2 hours ago
















  • Thank you! It seems as I have overseen quite a lot of good examples.
    – pcalc
    2 hours ago















Thank you! It seems as I have overseen quite a lot of good examples.
– pcalc
2 hours ago




Thank you! It seems as I have overseen quite a lot of good examples.
– pcalc
2 hours ago

















 

draft saved


draft discarded















































 


draft saved


draft discarded














StackExchange.ready(
function ()
StackExchange.openid.initPostLogin('.new-post-login', 'https%3a%2f%2fmath.stackexchange.com%2fquestions%2f2963260%2fsigned-measure-of-uncountable-set%23new-answer', 'question_page');

);

Post as a guest













































































Comments

Popular posts from this blog

Long meetings (6-7 hours a day): Being “babysat” by supervisor

Is the Concept of Multiple Fantasy Races Scientifically Flawed? [closed]

Confectionery